Careers360 Logo
Top Engineering Colleges in India with NIRF Rank Change

Proof of the Vector Triple Product - Practice Questions & MCQ

Edited By admin | Updated on Sep 18, 2023 18:34 AM | #JEE Main

Quick Facts

  • Vector Triple Product is considered one the most difficult concept.

  • 25 Questions around this concept.

Solve by difficulty

The vectors \vec{a}\; and \; \vec{b} are not perpendicular and \vec{c}\; and \; \vec{d} are two vectors satisfying: \vec{b}\; \times \; \vec{c}=\vec{b}\; \times \; \vec{d}and \vec{a}\cdot \vec{d}=0. Then the vector \vec{d} is equal to

\begin{array}{l}{\text { Let } \vec{a}=2 \hat{i}+\hat{j}-2 \hat{k} \text { and } \vec{b}=\hat{i}+\hat{j}} \\ {\text { Let } \vec{c} \text { be a vector such that }|\vec{c}-\vec{a}|=3,|(\vec{a} \times \vec{b}) \times \vec{c}|=3} \\ {\text { and the angle between } \vec{c} \text { and } \vec{a} \times \vec{b} \text { is } 30^{\circ} . \text { Then } \vec{a} . \vec{c} \text { is equal to }}\end{array}

Concepts Covered - 0

Vector Triple Product

For three  vectors \vec {\mathbf a},\;\vec{\mathbf b} and \vec {\mathbf c} vector triple product is defined as \vec {\mathbf a}\times\left ( \vec{\mathbf b}\times \vec{\mathbf c} \right ).
\vec {\mathbf a}\times\left ( \vec{\mathbf b}\times \vec{\mathbf c} \right )=\left ( \vec {\mathbf a}\cdot \vec {\mathbf c} \right )\cdot \vec {\mathbf b}\;-\;\left ( \vec {\mathbf a}\cdot \vec {\mathbf b} \right )\cdot \vec {\mathbf c}


\\\vec p=\vec { a}\times\left ( \vec{ b}\times \vec{ c} \right )\;\text{ is a vector perpendicular to }\;\vec a\;\;\text{and }\;\;\vec{b}\times \vec {c},\;\text{ but }\;\vec{ b}\times \vec{ c} \\\text{is a vector perpendicular to the plane of }\;\vec b\;\;\text{and}\;\;\vec c.\\\text {Hence, vector } \;\vec{p} \;\text { must lie in the plane of } \vec{b} \text { and } \vec{c} \text { . }\\ \;\;\;\;\;\;\;\;\;\;\;\;Let\,\,\vec p=\vec { a}\times\left ( \vec{ b}\times \vec{ c} \right )=l\;\vec {b}+m\;\vec{c}\;\;\;\;\;\;\;\;\;\;\;\;\;[l,m\;\text{are scalars}]\;\;\;\;\;\; \ldots \text{(i)}\\\text {Taking the dot product of eq (i) with } \vec{a}, \text { we get } \\\mathrm{\;\;\;\;\;\;\;\;\;\;\;\;}\vec{p} \cdot \vec{a}=l(\vec{a} \cdot \vec{b})+m(\vec{a} \cdot \vec{c})

\\\mathrm{\;\;\;\;\;\;\;\;}\begin{bmatrix} \because \;\vec { a}\times\left ( \vec{ b}\times \vec{ c} \right )\;\text{is}\;\perp\;\vec a\\ \therefore \vec { a}\times\left ( \vec{ b}\times \vec{ c} \right )\cdot \vec a=0 \end{bmatrix}\\\\\text{Therefore,}\\\mathrm{\Rightarrow \;\;\;\;\;\;\;\;\;}\vec{p} \cdot \vec{a}=0 \\\mathrm{\Rightarrow \;\;\;\;\;\;\;\;\;}l(\vec{a} \cdot \vec{b})=-m(\vec{a} \cdot \vec{c})\\\Rightarrow \;\;\;\;\;\;\;\frac{l}{\vec a \cdot\vec c}=\frac{-m}{\vec{a} \cdot \vec{b}}=\lambda\\\mathrm{{\Rightarrow \;\;\;\;\;\;\;\;\;}}l=\lambda\left ( \vec a\cdot \vec c \right )\\\mathrm{{and \;\;\;\;\;}}m=-\lambda\left ( \vec a\cdot \vec b \right )\\\text {Substituting the value of } l \text { and } m \text { in } \mathrm{Eq} \text { . (i), we get }\\\mathrm{\;\;\;}\vec{a} \times(\vec{b} \times \vec{c})=\lambda\left [ (\vec{a} \cdot \vec{c}) \vec{b}-(\vec{a}\cdot \vec{b}) \vec{c} \right ]

Here, the value of λ can be determined by taking specific values of \vec{a},\;\vec{b} and \vec{c}.

 

The simplest way to determine λ is by taking specific vectors \vec{a}=\hat{i}, \vec{b}=\hat{i}, \vec{c}=\hat{j}.

We have,

\\\mathrm{\;\;\;\;\;\;\;\;}\vec{a} \times(\vec{b} \times \vec{c})=\lambda\left [ (\vec{a} \cdot \vec{c}) \vec{b}-(\vec{a}\cdot \vec{b}) \vec{c} \right ]\\\\\mathrm{\;\;\;\;\;\;\;\;\;}\hat{i} \times(\hat{i} \times \hat{j})=\lambda \left [ (\hat{i} \cdot \hat{j}) \hat{i}-(\hat{i} \cdot \hat{i}) \hat{j} \right ]\\\\\mathrm{\;\;\;\;\;\;\;\;\;}\hat{i} \times\hat k =\lambda \left [ (0) \hat{i}-(\1) \hat{j} \right ]\;\Rightarrow \;-\hat j=-\lambda \hat j\\\\\therefore \;\;\;\;\;\lambda=1\\\text{Hence,}\\\mathrm{\;\;\;\;\;\;\;}\vec{a} \times(\vec{b} \times \vec{c})=(\vec{a} \cdot \vec{c}) \vec{b}-(\vec{a} \cdot \vec{b}) \vec{c}

 

NOTE:

\\1.\;\;\;\vec {\mathbf a}\times\left ( \vec{\mathbf b}\times \vec{\mathbf c} \right )=\left ( \vec {\mathbf a}\cdot \vec {\mathbf c} \right )\cdot \vec {\mathbf b}\;-\;\left ( \vec {\mathbf a}\cdot \vec {\mathbf b} \right )\cdot \vec {\mathbf c}\\\\\mathrm{\;\;\;\;\;} \left ( \vec {\mathbf a}\times \vec {\mathbf b} \right )\times\vec {\mathbf c}= \left ( \vec {\mathbf c}\cdot \vec {\mathbf a} \right )\cdot \vec {\mathbf b}\;-\;\left ( \vec {\mathbf c}\cdot \vec {\mathbf b} \right )\cdot \vec {\mathbf a} \\\\2.\;\;\text{In general}\;\;\vec {\mathbf a}\times\left ( \vec{\mathbf b}\times \vec{\mathbf c} \right )\neq\left ( \vec {\mathbf a}\times \vec {\mathbf b} \right )\times\vec {\mathbf c}\\\\\text{\;\;\;\;\;If }\;\;\vec {\mathbf a}\times\left ( \vec{\mathbf b}\times \vec{\mathbf c} \right )=\left ( \vec {\mathbf a}\times \vec {\mathbf b} \right )\times\vec {\mathbf c}\;\;\text{then the vectors }\vec a\;\;\text{and}\;\;\vec{c}\\\mathrm{\;\;\;\;\;are\;collinear}.

"Stay in the loop. Receive exam news, study resources, and expert advice!"

Get Answer to all your questions

Back to top